Complex Numbers 5 Question 4

4. Let $z=\cos \theta+i \sin \theta$. Then, the value of $\sum _{m=1}^{15} \operatorname{Im}\left(z^{2 m-1}\right)$ at $\theta=2^{\circ}$ is

(2009)

(a) $\frac{1}{\sin 2^{\circ}}$

(b) $\frac{1}{3 \sin 2^{\circ}}$

(c) $\frac{1}{2 \sin 2^{\circ}}$

(d) $\frac{1}{4 \sin 2^{\circ}}$

Show Answer

Answer:

Correct Answer: 4. (d)

Solution:

  1. Given that, $z=\cos \theta+i \sin \theta=e^{i \theta}$

$$ \begin{aligned} \therefore \sum _{\mu=1}^{15} I \mu\left(\zeta^{2 \mu-1}\right) & =\sum _{\mu=1}^{15} I \mu\left(\varepsilon^{(\theta}\right)^{2 \mu-1}=\sum _{\mu=1}^{15} I \mu \varepsilon^{l(2 \mu-1) \theta} \\ & =\sin \theta+\sin 3 \theta+\sin 5 \theta+\ldots+\sin 29 \theta \\ & =\frac{\sin \frac{\theta+29 \theta}{2} \sin \frac{15 \times 2 \theta}{2}}{\sin \frac{2 \theta}{2}} \\ & =\frac{\sin (15 \theta) \sin (15 \theta)}{\sin \theta}=\frac{1}{4 \sin 2^{\circ}} \end{aligned} $$



जेईई के लिए मॉक टेस्ट

एनसीईआरटी अध्याय वीडियो समाधान

दोहरा फलक